RD Sharma solution class 7 chapter 2 Fractions Exercise Objective Type Questions

Objective Type Questions

Page-2.26

Question 1:

Mark the correct alternative in each of the following:

If a fraction ab is a lowest terms, then HCF of a and b is

(a) a                                       (b) b                                       (c) 1                                       (d) ab                                 

Answer 1:


We know that a fraction is in its lowest terms if its numerator and denominator have no common factor other than 1.

Thus, if the fraction ab is in its lowest terms, then the HCF of a and b is 1.

Hence, the correct answer is option (c). 

Question 2:

Mark the correct alternative in each of the following:

The fraction 8498 in its lowest terms is

(a) 4249                                       (b) 1214                                       (c) 67                                       (d) 37                                 

Answer 2:


Factors of 84: 1, 2, 3, 4, 6, 7, 12, 14, 21, 28, 42, 84

Factors of 98: 1, 2, 7, 14, 49, 98

Common factors of 84 and 98: 1, 2, 14

∴ HCF of 84 and 98 = 14

Now,

8498=84÷1498÷14=67             (Dividing numerator and denominator by the HCF of 84 and 98 i.e. 14)

Hence, the correct answer is option (c).

Question 3:

Mark the correct alternative in each of the following:

Which of the following is a vulgar fraction?

(a) 710                                       (b) 131000                                       (c) 2910                                       (d) 79                                                                 

Answer 3:


The fractions with denominator not equal to 10, 100, 1000 etc are called vulgar fractions.

Thus, the fraction 79 is a vulgar fraction.

Hence, the correct answer is option (d).

Question 4:

Mark the correct alternative in each of the following:

Which of the following fraction is an irreducible (or in its lowest terms)?

(a) 91104                                       (b) 105112                                       (c) 5185                                       (d) 4383                                                    

Answer 4:

We know that a fraction is irreducible (or is in its lowest terms) if the HCF of its numerator and denominator is 1.

Consider the fraction 91104.

HCF of 91 and 104 = 13 ≠ 1

So, the fraction 91104 is reducible.

Consider the fraction 105112.

HCF of 105 and 112 = 7 ≠ 1

So, the fraction 105112 is reducible.

Consider the fraction 5185.

HCF of 51 and 85 = 17 ≠ 1

So, the fraction 5185 is reducible.

Now,

Consider the fraction 4383.

HCF of 43 and 83 = 1

So, the fraction 4383 is irreducible (or is in its lowest terms).

Hence, the correct answer is option (d).

Question 5:

Mark the correct alternative in each of the following:

Which of the following is a proper fraction?

(a) 1317                                       (b) 1713                                       (c) 125                                       (d) 134                                                                  

Answer 5:


A fraction whose numerator is less than the denominator is called a proper fraction.

The numerator in each of the fractions 1713, 125, 134=74 is more than the denominator, so these fractions are improper fractions.

The numerator of the fraction 1317 is less than the denominator, so this fraction is a proper fraction.

Hence, the correct answer is option (a).

Question 6:

Mark the correct alternative in each of the following:

The reciprocal of the fraction 235 is

(a) 253                                       (b) 135                                       (c) 513                                       (d) 225                                 

Answer 6:

The reciprocal of a non-zero fraction ab is the fraction ba.

235=2×5+35=135

Now,

Reciprocal of the fraction 135513

∴ Reciprocal of the fraction 235513

Hence, the correct answer is option (c).

Question 7:

Mark the correct alternative in each of the following:

413-213=

(a) 213                                      (b) 2                                       (c) 313                                      (d) 12                                 

Answer 7:


413-213=133-73=13-73=63=2

Hence, the correct answer is option (b).

Question 8:

Mark the correct alternative in each of the following:

235÷57=

(a) 137                                      (b) 1325                                       (c) 9125                                      (d) 2591                                 

Answer 8:


235÷57=135÷57=135×75=13×75×5=9125

Hence, the correct answer is option (c).

Question 9:

Mark the correct alternative in each of the following:

By what number should 134 be divided to get 212?

(a) 37                                      (b) 125                                       (c) 710                                      (d) 137

Answer 9:


Let the required number be x.

Now,

134÷x=21274×1x=52x=74×25x=7×12×5x=710

Thus, the required number is 710.

Hence, the correct answer is option (c).

Question 10:

Mark the correct alternative in each of the following:

By what number 435 be multiplied to get 237?

(a) 39135                                      (b) 8591                                       (c) 9185                                      (d) None of these                                

Answer 10:


Product of two numbers = 237=177

One of the numbers = 435=235

∴ Other number = Product of two numbers ÷ One of the numbers

                          =177÷235=177×523=17×57×23=85161

Hence, the correct answer is option (d).

Page-2.27

Question 11:

Mark the correct alternative in each of the following:

514-313=

(a) 1223                                      (b) 2                                       (c) 11112                                      (d) 1112   

Answer 11:


514-313=214-103=21×34×3-10×43×4                        LCM of 3 and 4 is 12=6312-4012
=63-4012=2312=11112

Hence, the correct answer is option (c).

Question 12:

Mark the correct alternative in each of the following:

The fraction equivalent to 123 is

(a) 103                                      (b) 35                                       (c) 106                                      (d) 610  

Answer 12:


The given fraction is 123=53.

We know that if ab and cd are two equivalent fractions, then

a×d=b×c 

Now,

5×6=3×10

So, the fractions 53 and 106 are equivalent fractions.

Thus, the fraction equivalent to 123 is 106.

Hence, the correct answer is option (c).

Question 13:

Mark the correct alternative in each of the following:

By what number 945 be multiplied to get 42?

(a) 307                                      (b) 730                                       (c) 417                                      (d) 437   

Answer 13:


Product of two numbers = 42

One of the numbers = 945=495

∴ Other number = Product of two numbers ÷ One of the numbers

                          =42÷495=421×549=6×51×7=307

Hence, the correct answer is option (a).

Question 14:

Mark the correct alternative in each of the following:

Which of the following statements is true?

(a) 712<421                                      (b) 712=421                                       (c) 712>421                                      (d) None of these   

Answer 14:


Consider the fractions 712 and 421.

Prime factorisation of 12 = 2 × 2 × 3

Prime factorisation of 21 = 3 × 7

∴ LCM of 12 and 21 = 2 × 2 × 3 × 7 = 84

Firstly, convert the fractions to equivalent fractions with denominator 84.

712=7×712×7=4984421=4×421×4=1684

Now,

49 > 16

4984>1684712>421

Hence, the correct answer is option (c).

Question 15:

Mark the correct alternative in each of the following:

Which one of the following is the correct statement?

(a) 34<23<1215                  (b) 23<34<1215                  (c) 23<1215<34                  (d) 1215<23<34  

Answer 15:


Consider the fractions 34, 23 and 1215.

LCM of 4, 3 and 15 = 60

Firstly, convert the fractions into equivalent fractions with denominator 60.

34=3×154×15=456023=2×203×20=40601215=12×415×4=4860

Now,

40 < 45 <  48

4060<4560<486023<34<1215

Hence, the correct answer is option (b).

Question 16:

Mark the correct alternative in each of the following:

Which of the following fractions lies between 23 and 57?

(a) 34                                      (b) 45                                       (c) 56                                      (d) None of these

Answer 16:


Consider the fractions 23, 57, 34, 45 and 56.

LCM of 3, 4, 5, 6 and 7 = 420

Firstly, convert the fractions into equivalent fractions with denominator 420.

23=2×1403×140=28042057=5×607×60=30042034=3×1054×105=31542045=4×845×84=33642056=5×706×70=350420

Now,

280 < 300 < 315 < 336 < 350

280420<300420<315420<336420<35042023<57<34<45<56

Thus, none of the fractions 34, 45, 56 lies between the fractions 23 and 57.

Hence, the correct answer is option (d).

Question 17:

Mark the correct alternative in each of the following:

Which one of the following is true?

(a) 12<913<34<1217                                                     (b) 34<913<12<1217                                      

(c) 12<34<913<1217                                                     (d) 12<913<1217<34   

Answer 17:


Consider the fractions 12, 913, 34 and 1217.

LCM of 2, 4, 13 and 17 = 884

Firstly, convert the fractions into equivalent fractions with denominator 884.

12=1×4422×442=442884913=9×6813×68=61288434=3×2214×221=6638841217=12×5217×52=624884

Now,

442 < 612 < 624 < 663

442884<612884<624884<66388412<913<1217<34

Hence, the correct answer is option (d).

Question 18:

Mark the correct alternative in each of the following:

The smallest of the fractions 23,47,811 and 59 is

(a) 23                                      (b) 47                                       (c) 811                                      (d) 59   

Answer 18:


The given fractions are 23,47,811 and 59.

LCM of 3, 7, 9 and 11 = 693

Firstly, convert the fractions into equivalent fractions with denominator 693.

23=2×2313×231=46269347=4×997×99=396693811=8×6311×63=50469359=5×779×77=385693

Now,

385 < 396 < 462 < 504

385693<396693<462693<50469359<47<23<811

Thus, the smallest of the given fractions is 59.

Hence, the correct answer is option (d).

Question 19:

Mark the correct alternative in each of the following:

9×-13×-3×-19=

(a) 1                                      (b) −1                                       (c) −3                                      (d) 3  

Answer 19:


Since the number of negative terms in the product is odd. Therefore, their product is negative.

9×-13×-3×-19=9×-19×-13×-3=-9×19×13×3=-1×1=-1

Hence, the correct answer is option (b).

Question 20:

Mark the correct alternative in each of the following:

Which of the following is correct?

(a) 23<35<1115                  (b) 35<23<1115                  (c) 1115<35<23                  (d) 35<1115<23  

Answer 20:

Consider the fractions 23, 35 and 1115.

LCM of 3, 5 and 15 = 15

Firstly, convert the fractions into equivalent fractions with denominator 15.

23=2×53×5=101535=3×35×3=915

Now,

9 < 10 < 11

915<1015<111535<23<1115

Hence, the correct answer is option (b).

Page-2.28

Question 21:

Mark the correct alternative in each of the following:

Which is the smallest of the following fractions?

(a) 49                                 (b) 25                                 (c) 37                                  (d) 14  

Answer 21:

Consider the fractions 49, 25, 37 and 14.

LCM of 4, 5, 7 and 9 = 1260

Firstly, convert the fractions into equivalent fractions with denominator 1260.

49=4×1409×140=560126025=2×2525×252=504126037=3×1807×180=540126014=1×3154×315=3151260

Now,

315 < 504 < 540 < 560

3151260<5041260<5401260<560126014<25<37<49

Thus, the smallest fraction is 14.

Hence, the correct answer is option (d).

Question 22:

Mark the correct alternative in each of the following:

The difference between the greatest and the least fractions out of 67,78,89 and 910 is

(a) 310                            (b) 156                             (c) 140                            (d) 172  

Answer 22:


Consider the fractions 67,78,89  and 910.

LCM of 7, 8, 9 and 10 = 2520

Firstly, convert the fractions into equivalent fractions with denominator 2520.

67=6×3607×360=2160252078=7×3158×315=2205252089=8×2809×280=22402520910=9×25210×252=22682520

Now,

2160 < 2205 < 2240 < 2268

21602520<22052520<22402520<2268252067<78<89<910

So,

Greatest fraction = 910

Least fraction = 67

∴ Required difference

=910-67=22682520-21602520=2268-21602520=1082520

=108÷362520÷36                  HCF of 108 and 2520=36=370


Disclaimer: None of the options given in the question matches with the answer.

Question 23:

Mark the correct alternative in each of the following:

Which of the following fractions is greater than 34 and less than 56?

(a) 23                               (b) 12                               (c) 45                               (d) 910  

Answer 23:


Consider the fractions 34,56,23,12,45 and 910.

LCM of 2, 3, 4, 5, 6 and 10 = 60

Firstly, convert the fractions into equivalent fractions with denominator 60.

34=3×154×15=456056=5×106×10=506023=2×203×20=406012=1×302×30=306045=4×125×12=4860910=9×610×6=5460

Now,

30 < 40 < 45 < 48 < 50 < 54

3060<4060<4560<4860<5060<546012<23<34<45<56<910

Thus, the fraction 45 is greater than 34 and less than 56.

Hence, the correct answer is option (c).

Question 24:

Mark the correct alternative in each of the following:

Which of the following fractions is more than one-third?

(a) 2370                               (b) 205819                               (c) 2675                               (d) 118335  

Answer 24:


Let ab and cd be two fractions. Then, ab>cd if a×d>b×c.

Consider the fractions 2370 and 13.
23×3=691×70=7023×3<1×702370<13

Consider the fractions 205819 and 13.
205×3=6151×819=819205×3<1×819205819<13

Consider the fractions 2675 and 13.
26×3=781×75=7526×3>1×752675>13

Consider the fractions 118335 and 13.
118×3=3541×335=335118×3>1×335118335>13

Thus, the fractions 2675 and 118335 are more than the fraction 13.

Hence, the correct answers are options (c) and (d).


Disclaimer: There are two correct options in the question. One of the two options among (c) and (d) must be changed accordingly to get only one correct answer.

3 comments:

  1. I like it the answers are so easy and accurate to understand. I am thankful to myhelpertk.

    ReplyDelete
  2. I like it the answers are so easy and accurate to understand. I am thankful to myhelpertk.

    ReplyDelete
  3. I like it the answers are so easy and accurate to understand. I am thankful to myhelpertk.

    ReplyDelete

Contact Form

Name

Email *

Message *